Last visit was: 25 Apr 2024, 15:45 It is currently 25 Apr 2024, 15:45

Close
GMAT Club Daily Prep
Thank you for using the timer - this advanced tool can estimate your performance and suggest more practice questions. We have subscribed you to Daily Prep Questions via email.

Customized
for You

we will pick new questions that match your level based on your Timer History

Track
Your Progress

every week, we’ll send you an estimated GMAT score based on your performance

Practice
Pays

we will pick new questions that match your level based on your Timer History
Not interested in getting valuable practice questions and articles delivered to your email? No problem, unsubscribe here.
Close
Request Expert Reply
Confirm Cancel
SORT BY:
Date
Tags:
Show Tags
Hide Tags
User avatar
Senior Manager
Senior Manager
Joined: 29 Oct 2004
Posts: 456
Own Kudos [?]: 1498 [113]
Given Kudos: 0
Send PM
Most Helpful Reply
GMAT Club Verbal Expert
Joined: 13 Aug 2009
Status: GMAT/GRE/LSAT tutors
Posts: 6921
Own Kudos [?]: 63668 [21]
Given Kudos: 1774
Location: United States (CO)
GMAT 1: 780 Q51 V46
GMAT 2: 800 Q51 V51
GRE 1: Q170 V170

GRE 2: Q170 V170
Send PM
avatar
Intern
Intern
Joined: 12 Dec 2004
Posts: 2
Own Kudos [?]: 8 [8]
Given Kudos: 0
Send PM
General Discussion
User avatar
Manager
Manager
Joined: 20 Nov 2006
Posts: 112
Own Kudos [?]: 301 [3]
Given Kudos: 0
Send PM
Re: Gortland has long been narrowly self-sufficient in both [#permalink]
2
Kudos
1
Bookmarks
E is the answer. If the meat consumption increases, the grain consumption should come down, which will offset partially the production of meat. But if it doesn't happen, then the import becomes necessary
User avatar
Senior Manager
Senior Manager
Joined: 30 Nov 2006
Posts: 352
Own Kudos [?]: 891 [5]
Given Kudos: 0
Location: Kuwait
Concentration: Strategy - Finance
 Q49  V36
Send PM
Re: Gortland has long been narrowly self-sufficient in both [#permalink]
5
Kudos
Most answers are either out-of-scope or are not really an assumption made to derive to the conclusion, except E.

Prod: production, Cons: consumption

Must be: Grain Prod + Meat Prod = Meat Cons + Grain Cons
Grain Prod : SAME
Grain Cons: NO IDEA
Meat Prod: SAME
Meat Cons: UP

In order for the equation to be valid, grain consumption must decrease. Since the author suggest importing grain or meat or both, then he must've assumed that grain consumption will not decrease to balance the equation of production and consumption.

ANSWER: E
Retired Moderator
Joined: 16 Nov 2010
Posts: 909
Own Kudos [?]: 1173 [2]
Given Kudos: 43
Location: United States (IN)
Concentration: Strategy, Technology
Send PM
Re: Gortland has long been narrowly self-sufficient in both [#permalink]
2
Kudos
B and E are the ones to be considered. I chose E over B because on a second look at B,if the popultation growth is negated then we havt to consider both rise and fall in population, and these two may have different effects on the argument. For example, if population decreases while total meat consumed is same, the per capita consumption increases and if population groth increases while total meat consumed is same, the per capita consumption decreases.

E states that if both grains and fruits are consumed at an increasing rate, there will be a shortage making import of both necessary.
User avatar
Director
Director
Joined: 02 Sep 2012
Status:Far, far away!
Posts: 859
Own Kudos [?]: 4891 [1]
Given Kudos: 219
Location: Italy
Concentration: Finance, Entrepreneurship
GPA: 3.8
Send PM
Re: Gortland has long been narrowly self-sufficient in both [#permalink]
1
Kudos
lucasITA wrote:
i've a simple question:
the existence of a positive correlation between growth of per capita income and meat consumption is a valid assumption?

because this reasoning brings me to chose option C :(


We know that both the income and the meat consumption are rising, we can say that with an higher income their have more money to spend on meat as well. But this could be an inference, not an assumption.

Option C
(C) The per capita consumption of meat in Gortland is roughly the same across all income levels

is not an assumption (hence something necessary for the argument to be valid); the conclusion is "Gortland will soon have to import either grain or meat or both" and even if we negate C "the per capita cons is NOT the same" the argument still holds true.

Even if the richest consume more meat than the others (for example), still Gortland will have to import one thing or both. Keep in mind that the assumption is something necessary, so if you negate an option and still the conclusion is valid=>the option is not the assumption we're looking for.
Intern
Intern
Joined: 05 Feb 2014
Posts: 20
Own Kudos [?]: 12 [3]
Given Kudos: 74
GMAT 1: 700 Q45 V40
Send PM
Re: Gortland has long been narrowly self-sufficient in both [#permalink]
3
Kudos
LucyDang wrote:
Hi,

I agree that if consumption of grain decrease, there is no need for Gortland to import grain! But I still don't understand why there is no need of importing meat in Gortland if consumption of grain decrease and consumption of meat increase?


The argument clearly says that the production of meat depends on the amount of grain available. That means, if the consumption of grain decreases and thereby increasing the amount of grain available for meat production, meat production will ultimately increase. Hence, no need to import either grain or meat.

Hope it makes sense :-)
Manager
Manager
Joined: 14 Feb 2016
Posts: 54
Own Kudos [?]: 161 [2]
Given Kudos: 49
Send PM
Re: Gortland has long been narrowly self-sufficient in both [#permalink]
2
Kudos
Gortland has long been narrowly self-sufficient in both grain and meat. However, as per capita income in Gortland has risen toward the world average, per capita consumption of meat has also risen toward the world average, and it takes several pounds of grain to produce one pound of meat. Therefore, since per capita income continues to rise, whereas domestic grain production will not increase, Gortland will soon have to import either grain or meat or both.

Which of the following is an assumption on which the argument depends?

(A) The total acreage devoted to grain production in Gortland will soon decrease.
(B) Importing either grain or meat will not result in a significantly higher percentage of Gortlanders' incomes being spent on food than is currently the case.
(C) The per capita consumption of meat in Gortland is increasing at roughly the same rate across all income levels.
(D) The per capita income of meat producers in Gortland is rising faster than the per capita income of grain producers.
(E) People in Gortland who increase their consumption of meat will not radically decrease their consumption of grain.

Conclusion: Gortland will have to import either grain or meat or both.

Premise(s): As per capita income rises, so will per capita consumption of meat. It takes several pounds of grain to produce one pound of meat, and domestic grain production will not increase. So Gortland will have to import either grain or meat or both.

Prephrase: Let’s try to identify a possible assumption before answering the answer choices. It’s hard to identify one, so I think we’re working with a defender assumption. Let’s use the assumption negation technique and see which one weakens the argument the most. Using the assumption negation technique in the answer choices we can try and find the assumption on which the argument depends.

a) If grain production in Gortland will not soon decrease – Gortland will have to import either grain or meat or both. Thus, grain production in Portland will not increase, nor decrease, this keeps the premise intact and therefore doesn’t weaken the argument.
b) Importing either grain or meat will result in a significantly higher percentage of Gortlanders incomes being spent on food than is currently the case. Thus, food consumption will rise, therefore per capita consumption of meat is also likely to increase. Doesn’t weaken.
c) The per capita consumption of meat in Gortland is not increasing at roughly the same rate across all income levels. Thus different income levels have different levels of meat consumption. But it’s still increasing, so it still keeps the premises in check.
d) The per capita income of meat producers in Gortland is not rising faster than the per capita income of grain producers. The income of the meat producers themselves isn’t relevant. Therefore out of scope.
e) People in Portland who increase their consumption of meat will radically decrease their consumption of grain. If people in Portland decrease their consumption of grain, then Gortland will no longer have to import either grain or meat or both.

Therefore, answer choice E is correct. As it weakens the argument the most and is therefore the underlying assumption that holds the entire frigging thing together.
Senior Manager
Senior Manager
Joined: 22 Feb 2018
Posts: 355
Own Kudos [?]: 799 [1]
Given Kudos: 34
Send PM
Re: Gortland has long been narrowly self-sufficient in both [#permalink]
1
Kudos
nightblade354 Bunuel

In this thread,two questions are being discussed. Argument,question stem and OA are same for both questions but options are different.

1st Question
Gortland has long been narrowly self-sufficient in both grain and meat. However, as per capita income in Gortland has risen toward the world average, per capita consumption of meat has also risen toward the world average, and it takes several pounds of grain to produce one pound of meat. Therefore, since per capita income continues to rise, whereas domestic grain production will not increase, Gortland will soon have to import either grain or meat or both.

Which of the following is an assumption on which the argument depends?

(A) The total acreage devoted to grain production in Gortland will soon decrease.
(B) Importing either grain or meat will not result in a significantly higher percentage of Gortlanders' incomes being spent on food than is currently the case.
(C) The per capita consumption of meat in Gortland is increasing at roughly the same rate across all income levels.
(D) The per capita income of meat producers in Gortland is rising faster than the per capita income of grain producers.
(E) People in Gortland who increase their consumption of meat will not radically decrease their consumption of grain.

2 nd
Gortland has long been narrowly self-sufficient in both grain and meat. However, as per capita income in Gortland has risen toward the world average, per capita consumption of meat has also risen toward the world average, and it takes several pounds of grain to produce one pound of meat. Therefore, since per capita income continues to rise, whereas domestic grain production will not increase, Gortland will soon have to import either grain or meat or both

Which of the following is an assumption on which the argument depends?

(A) The total acreage devoted to grain production in Gortland will not decrease substantially.
(B) The population of Gortland has remained relatively constant during the country's years of growing prosperity.
(C) The per capita consumption of meat in Gortland is roughly the same across all income levels.
(D) In Gortland, neither meat nor grain is subject to government price controls.
(E) People in Gortland who increase their consumption of meat will not radically decrease their consumption of grain.

Is possible to have seperate threads for these 2 questions?
Current Student
Joined: 31 Jul 2017
Status:He came. He saw. He conquered. -- Going to Business School -- Corruptus in Extremis
Posts: 1734
Own Kudos [?]: 5740 [1]
Given Kudos: 3054
Location: United States (MA)
Concentration: Finance, Economics
Send PM
Re: Gortland has long been narrowly self-sufficient in both [#permalink]
1
Kudos
Expert Reply
Princ,

The topics have been split and the second question has been posted here.

I understand the confusion, so good job pointing it out.

-- Mod Nightblade
Intern
Intern
Joined: 10 Sep 2018
Posts: 18
Own Kudos [?]: 6 [0]
Given Kudos: 6
Send PM
Re: Gortland has long been narrowly self-sufficient in both [#permalink]
qhoc0010 wrote:
Gortland has long been narrowly self-sufficient in both grain and meat. However, as per capita income in Gortland has risen toward the world average, per capita consumption of meat has also risen toward the world average, and it takes several pounds of grain to produce one pound of meat. Therefore, since per capita income continues to rise, whereas domestic grain production will not increase, Gortland will soon have to import either grain or meat or both

Which of the following is an assumption on which the argument depends?

(A) The total acreage devoted to grain production in Gortland will soon decrease.
(B) Importing either grain or meat will not result in a significantly higher percentage of Gortlanders' incomes being spent on food than is currently the case.
(C) The per capita consumption of meat in Gortland is increasing at roughly the same rate across all income levels.
(D) The per capita income of meat producers in Gortland is rising faster than the per capita income of grain producers.
(E) People in Gortland who increase their consumption of meat will not radically decrease their consumption of grain.


'A very similar question exists in Verbal OG: https://gmatclub.com/forum/gortland-has ... 12544.html


Official Guide 12 Question

GMAT Official Guide 12

Question: 28
Page: 37
Difficulty: 600

Find All Official Guide Questions

Video Explanations:


Conclusion : Gortland will soon have to import either grain or meat or both

(A) The total acreage devoted to grain production in Gortland will soon decrease.
Irrelevant to the conclusion; eliminate
(B) Importing either grain or meat will not result in a significantly higher percentage of Gortlanders' incomes being spent on food than is currently the case.
Irrelevant to the conclusion; eliminate
(C) The per capita consumption of meat in Gortland is increasing at roughly the same rate across all income levels.
If it's increasing across all income levels or not is irrelevant. We're concerned and are already told about the net per capita increase; Eliminate
(D) The per capita income of meat producers in Gortland is rising faster than the per capita income of grain producers.
Income is irrelevant to the argument; Eliminate
(E) People in Gortland who increase their consumption of meat will not radically decrease their consumption of grain.
Correct; If people will radically decrease their consumption of grain then the conclusion of importing grain/meat or both falls apart.
User avatar
Non-Human User
Joined: 01 Oct 2013
Posts: 17221
Own Kudos [?]: 848 [0]
Given Kudos: 0
Send PM
Re: Gortland has long been narrowly self-sufficient in both [#permalink]
Hello from the GMAT Club VerbalBot!

Thanks to another GMAT Club member, I have just discovered this valuable topic, yet it had no discussion for over a year. I am now bumping it up - doing my job. I think you may find it valuable (esp those replies with Kudos).

Want to see all other topics I dig out? Follow me (click follow button on profile). You will receive a summary of all topics I bump in your profile area as well as via email.
GMAT Club Bot
Re: Gortland has long been narrowly self-sufficient in both [#permalink]
Moderators:
GMAT Club Verbal Expert
6921 posts
GMAT Club Verbal Expert
238 posts
CR Forum Moderator
832 posts

Powered by phpBB © phpBB Group | Emoji artwork provided by EmojiOne